elchapo8362
elchapo8362
16.12.2021 • 
Mathematics

Does a value of k exist such that the following limit exists? if so, find the value of k and the corresponding limit. if not, explain why not lim x->3 2x^2 + kx -9/ x^2- 4x +3

Solved
Show answers

Ask an AI advisor a question